Skip to main content

Section 5.2 Multiple Choice

Exercises 5.2.1 Exercises

For each of the following, circle only one answer.

1.

If \(h(x)=\ln (1-x^2)\) where \(-1\lt x\lt 1\text{,}\) then \(h^\prime(x)=\)

A. \(\ds \frac{1}{1-x^2}\text{,}\) B. \(\ds \frac{1}{1+x}-\frac{1}{1-x}\text{,}\) C. \(\ds \frac{2}{1-x^2}\text{,}\) D. None of these.

Answer
B.
2.

The derivative of \(f(x)=x^2\tan x\) is

A. \(2x\sec^2x\text{,}\) B. \(2x\tan x+x^2\cot x\text{,}\) C. \(2x\tan x+(x\sec x)^2\text{,}\) D. None of these.

Answer
C.
3.

If \(\cosh y=x+x^3y\text{,}\) then at the point \((1,0)\) we have \(y^\prime\)

A. \(0\text{,}\) B. \(3\text{,}\) C. \(-1\text{,}\) D. Does not exist.

Answer
C.
4.

The derivative of \(\ds g(x)=e^{\sqrt{x}}\) is

A. \(e^{\sqrt{x}}\text{,}\) B. \(\sqrt{x}e^{\sqrt{x}-1}\text{,}\) C. \(\frac{0.5e^{\sqrt{x}}}{\sqrt{x}}\text{,}\) D. None of these.

Answer
C.

For each of the following, circle only one answer.

5.

Suppose \(y^{\prime\prime}+y=0\text{.}\) Which of the following is a possibility for \(y=f(x)\text{.}\)

A. \(y=\tan x\text{,}\) B. \(y=\sin x\text{,}\) C. \(y=\sec x\text{,}\) D. \(y=1/x\text{,}\) E. \(y=e^x\)

Answer
B.
6.

Which of the following is \(\ds \arcsin \left( \sin \left( \frac{3\pi }{4}\right) \right)\text{?}\)

A. \(0\text{,}\) B. \(\ds \frac{\pi }{4}\text{,}\) C. \(\ds -\frac{\pi }{4}\text{,}\) D. \(\ds \frac{5\pi }{4}\text{,}\) E. \(\ds \frac{3\pi }{4}\)

Answer
C. The range of \(y=\arcsin x\) is \(\ds\left[ -\frac{\pi }{2}, \frac{\pi }{2}\right]\text{.}\)
7.

Let \(f(x)\) be a continuous function on \([a,b]\) and differentiable on \((a,b)\) such that \(f(b)=10\text{,}\) \(f(a)=2\text{.}\) On which of the following intervals \([a,b]\) would the Mean Value Theorem guarantee a \(c\in (a,b)\) such that \(f'(c)=4\text{.}\)

A. \([0,4]\text{,}\) B. \([0,3]\text{,}\) C. \([2,4]\text{,}\) D. \([1,10]\text{,}\) E. \((0,\infty )\)

Answer
C. \(\ds \frac{10-2}{4-2}=4\text{.}\)
8.

Let \(P(t)\) be the function which gives the population as a function of time. Assuming that \(P(t)\) satisfies the natural growth equation, and that at some point in time \(t_0\text{,}\) \(P(t_0)=500\text{,}\) \(P'(t_0)=1000\text{,}\) find the growth rate constant \(k\text{.}\)

A. \(\ds -\frac{1}{2}\text{,}\) B. \(\ds \ln \left( \frac{1}{2}\right)\text{,}\) C. \(\ds \frac{1}{2}\text{,}\) D. \(2\text{,}\) E. \(\ln 2\)

Answer
C. Use \(\ds \frac{dP}{dt}=kP\text{.}\)
9.

Suppose that \(f\) is continuous on \([a,b]\) and differentiable on \((a,b)\text{.}\) If \(f^\prime(x)>0\) on \((a,b)\text{.}\) Which of the following is necessarily true?

A. \(f\) is decreasing on \([a,b]\text{,}\)

B. \(f\) has no local extrema on \((a,b)\text{,}\)

C. \(f\) is a constant function on \((a,b)\text{,}\)

D. \(f\) is concave up on \((a,b)\text{,}\)

E. \(f\) has no zero on \((a,b)\)

Answer
B. \(f\) is increasing.

For each of the following, circle only one answer.

10.

The equation \(x^5+10x+3=0\) has

A. no real roots, B. exactly one real root, C. exactly two real roots, D. exactly three real roots, E. exactly five real roots

Answer
B. Consider \(f(x)=x^5+10x+3\) and its first derivative.
11.

The value of \(\cosh (\ln 3)\) is

A. \(\ds \frac{1}{3}\text{,}\) B. \(\ds \frac{1}{2}\text{,}\) C. \(\ds \frac{2}{3}\text{,}\) D. \(\ds \frac{4}{3}\text{,}\) E. \(\ds \frac{5}{3}\)

Answer
E. \(\ds \cosh (\ln 3)=\frac{3+\frac{1}{3}}{2}\text{.}\)
12.

The function \(f\) has the property that \(f(3)=2\) and \(f'(3)=4\text{.}\) Using a linear approximation to \(f\) near \(x=3\text{,}\) an approximation to \(f(2.9)\) is

A. \(1.4\text{,}\) B. \(1.6\text{,}\) C. \(1.8\text{,}\) D. \(1.9\text{,}\) E. \(2.4\)

Answer
B. \(f(2.9)\approx 2+4(2.9-3)\text{.}\)
13.

Suppose \(F\) is an antiderivative of \(f(x)=\sqrt[3]{x}\text{.}\) If \(\ds F(0)=\frac{1}{4}\text{,}\) then \(F(1)\) is

A. \(-1\text{,}\) B. \(\ds -\frac{3}{4}\text{,}\) C. \(0\text{,}\) D. \(\ds \frac{3}{4}\text{,}\) E. \(1\)

Answer
E. \(F(x)=\frac{3}{4}x^{\frac{4}{3}}+\frac{1}{4}\text{.}\)
14.

Suppose \(f\) is a function such that \(f'(x)=4x^3\) and \(f"(x)=12x^2\text{.}\) Which of the following is true?

A. \(f\) has a local maximum at \(x=0\) by the first derivative test

B. \(f\) has a local minimum at \(x=0\) by the first derivative test.

C. \(f\) has a local maximum at \(x=0\) by the second derivative test.

D. \(f\) has a local minimum at \(x=0\) by the second derivative test.

E. \(f\) has an inflection point at \(x=0\)

Answer
B.

Circle clearly your answer to the following 10 multiple choice question.

15.

Evaluate \(\ds \frac{d}{dx}\sin (x^2)\)

A. \(2x\cos (x^2)\text{,}\) B. \(2x\sin (x^2)\text{,}\) C. \(2x\cos (x)\text{,}\) D. \(2x\cos (2x)\text{,}\) E. \(2x\cos (2x)\)

Answer
A.
16.

Evaluate \(\ds \lim _{x\to 0^+}\frac{\ln x}{x}\)

A. \(0\text{,}\) B. \(\infty\text{,}\) C. \(1\text{,}\) D. \(-1\text{,}\) E. none of the above

Answer
E. \(\ds \lim _{x\to 0^+}\frac{\ln x}{x}=-\infty\text{.}\)
17.

Evaluate \(\ds \lim _{x\to 0^+}\frac{1-e^x}{\sin x}\)

A. \(1\text{,}\) B. \(-1\text{,}\) C. \(0\text{,}\) D. \(\infty\text{,}\) E. \(\sin e\)

Answer
B. Use L'Hospital's rule.
18.

The circle described by the equation \(x^2+y^2-2x-4=0\) has center \((h,k)\) and radius \(r\text{.}\) The values of \(h\text{,}\) \(k\text{,}\) and \(r\) are

A. \(0\text{,}\) \(1\text{,}\) and \(\sqrt{5}\text{,}\) B. \(1\text{,}\) \(0\text{,}\) and \(5\text{,}\) C. \(1\text{,}\) \(0\text{,}\) and \(\sqrt{5}\text{,}\) D. \(-1\text{,}\) \(0\text{,}\) and \(5\text{,}\) E. \(-1\text{,}\) \(0\text{,}\) and \(\sqrt{5}\)

Answer
C. \((x-1)^2+y^2=5\text{.}\)
19.

The edge of the cube is increasing at a rate of 2 cm/hr. How fast is the cube's volume changing when its edge is \(\sqrt{x}\) cm in length?

A. 6 cm\(^3\)/hr, B. 12 cm\(^3\)/hr, C. \(3\sqrt{2}\) cm\(^3\)/hr, D. \(6\sqrt{2}\) cm\(^3\)/hr, E. none of the above

Answer
B. \(\ds \frac{dV}{dt}=3x^2\frac{dx}{dt}\text{.}\)
20.

Given the polar equation \(r=1\text{,}\) find \(\ds \frac{dy}{dx}\)

A. \(\cot \theta\text{,}\) B. \(-\tan \theta\text{,}\) C. \(0\text{,}\) D. \(1\text{,}\) E. \(-\cot \theta\)

Answer
E. \(\ds \frac{dy}{dt}=\frac{\frac{dy}{d\theta }}{\frac{dy}{d\theta }}\text{.}\)
21.

Let \(A(t)\) denote the amount of a certain radioactive material left after time \(t\text{.}\) Assume that \(A(0)=16\) and \(A(1)=12\text{.}\) How much time is left after time \(t=3\text{?}\)

A. \(\ds \frac{16}{9}\text{,}\) B. \(8\text{,}\) C. \(\ds \frac{9}{4}\text{,}\) D. \(\ds \frac{27}{4}\text{,}\) E. \(4\)

Answer
D. \(\ds A(t)=16\left( \frac{3}{4}\right) ^t\text{.}\)
22.

Which of the following statements is always true for a function \(f(x)\text{?}\)

  1. If \(f(x)\) is concave up on the interval \((a,b)\text{,}\) then \(f(x)\) has a local minimum \((a,b)\text{.}\)

  2. It is possible for \(y=f(x)\) to have an inflection point at \((a,f(a))\) even if \(f'(x)\) does not exist at\(x=a\text{.}\)

  3. It is possible for \((a,f(a))\) to be both a critical point and an inflection point of \(f(x)\text{.}\)

A. i. and ii.

B. only iii.

C. i., ii., and iii.

D. ii. and iii.

E. only i.

Answer
D. For (1) take \(f(x)=x^3\) on \((0,1)\text{.}\) For (2) take \(f(x)=\sqrt[3]{x}\text{.}\) For (3) take \(f(x)=x^4\text{.}\)
23.

Which of the following statements is always true for a function \(f(x)\text{?}\)

  1. If \(f(x)\) and \(g(x)\) are continuous at \(x=a\text{,}\) then \(\ds \frac{f(x)}{g(x)}\) is continuous at \(x=a\text{.}\)

  2. If \(f(x)+g(x)\) is continuous at \(x=a\) and \(f'(a)=0\text{,}\) then \(g(x)\) is continuous ta \(x=a\text{.}\)

  3. If \(f(x)+g(x)\) is differentiable at \(x=a\text{,}\) then \(f(x)\) and \(g(x)\) are both differentiable at \(x=a\text{.}\)

A. only i.

B. only ii.

C. only iii.

D. i. and ii.

E. ii. and iii.

Answer
B. For (1) take \(g(x)=0\text{.}\) For (3) take \(f(x)=|x|\text{,}\) \(g(x)=-|x|\text{,}\) and \(a=0\text{.}\)
24.

The slant asymptote of the function \(\ds f(x)=\frac{x^2+3x-1}{x-1}\) is

A. \(y=x+4\text{,}\) B. \(y=x+2\text{,}\) C. \(y=x-2\text{,}\) D. \(y=x-4\text{,}\) E. none of the above

Answer
A.

This is a multiple choice question. No explanation is required.

25.

The derivative of \(\ds g(x)=e^{\sqrt{x}}\) is

A. \(\sqrt{x}e^{\sqrt{x}-1}\text{,}\)

B. \(2e^{\sqrt{x}}x^{-0.5}\text{,}\)

C. \(\ds \frac{0.5e^{\sqrt{x}}}{\sqrt{x}}\text{,}\)

D. \(e^{\sqrt{x}}\text{,}\)

E. None of these

Answer
C.
26.

If \(\cosh y=x+x^3y\text{,}\) then at the point \((1,0)\) \(y^\prime =\)

A. \(0\text{,}\)

B. \(-1\text{,}\)

C. \(1\text{,}\)

D. \(3\text{,}\)

E. Does not exist

Answer
B. Note \(\ds y^\prime \sinh y=1+3x^2y+x^3y^\prime\text{.}\)
27.

An antiderivative of \(f(x)=x-\sin x+e^x\) is

A. \(1-\cos x +e^x\text{,}\)

B. \(x^2+\ln x-\cos x\text{,}\)

C. \(\ds 0.5x^2+e^x-\cos x\text{,}\)

D. \(\cos x +e^x+0.5x^2\text{,}\)

E. None of these

Answer
D.
28.

If \(h(x)=\ln (1-x^2)\) where \(-1\lt x\lt 1\text{,}\) then \(h^\prime (x)=\)

A. \(\ds \frac{1}{1-x^2}\text{,}\)

B. \(\ds \frac{1}{1+x}+\frac{1}{1-x}\text{,}\)

C. \(\ds \frac{2}{1-x^2}\text{,}\)

D. \(\ds \frac{1}{1+x}-\frac{1}{1-x}\text{,}\)

E. None of these

Answer
D.
29.

The linear approximation to \(f(x)=\sqrt[3]{x}\) at \(x=8\) is given by

A. \(2\text{,}\)

B. \(\ds \frac{x+16}{12}\text{,}\)

C. \(\ds \frac{1}{3x^{2/3}}\text{,}\)

D. \(\ds \frac{x-2}{3}\text{,}\)

E. \(\ds \sqrt[3]{x}-2\)

Answer
B.

This is a multiple choice question. No explanation is required.

30.

If a function \(f\) is continuous on the closed interval \([a,b]\) and differentiable on the open interval \((a,b)\text{,}\) then there exists \(c\in (a,b)\) such that \(\ds f(b)-f(a)=f^\prime(c)(b-a)\) is:

A. The Extreme Value Theorem,

B. The Intermediate Value Theorem,

C. The Mean Value Theorem,

D. Rolle's Theorem,

E. None of these

Answer
C.
31.

If \(f\) is continuous function on the closed interval \([a,b]\text{,}\) and \(N\) is a number between \(f(a)\) and \(f(b)\text{,}\) then there is \(c\in [a,b]\) such that \(f(c)=N\) is:

A. Fermat's Theorem

B. The Intermediate Value Theorem

C. The Mean Value Theorem

D. Rolle's Theorem

E. The Extreme Value Theorem

Answer
B.
32.

If \(f\) is continuous function on the open interval \((a,b)\) then \(f\) attains an absolute maximum value \(f(c)\) and an absolute minimum value \(f(d)\) at some numbers \(c,d\in (a,b)\) is:

A. The Extreme Value Theorem,

B. The Intermediate Value Theorem,

C. The Mean Value Theorem,

D. Rolle's Theorem,

E. None of these

Answer
E.
33.

A function \(f\) is continuous at a number \(a\) …

A. … if \(f\) is defined at \(a\text{,}\)

B. … if \(\ds \lim_{x\to a}\frac{f(x)-f(a)}{x-a}\) exists,

C. … if \(\ds \lim_{x\to a} f(x)\) exists,

D. … if \(f\) is anti-differentiable at \(a\text{,}\)

E. … if \(\ds \lim_{x\to a} f(x)=f(a)\)

Answer
E.
34.

A function \(f\) is differentiable at a number \(a\) …

A. … if \(\ds \lim_{x\to a} f(x)=f(a)\text{,}\)

B. … if \(\ds \lim_{x\to a}\frac{f(x)-f(a)}{x-a}\) exists,

C. … if \(f\) is defined at \(a\text{,}\)

D. … if \(f\) is continuous at \(a\text{,}\)

E. … if we can apply the Intermediate Value Theorem

Answer
B.
35.

An antiderivative of a function \(f\) …

A. … is a function \(\ds F\) such that \(F^\prime(x)=f(x)\text{,}\)

B. … is a function \(\ds F\) such that \(F(x)=f^\prime(x)\text{,}\)

C.… is a function \(\ds F\) such that \(F^\prime(x)=f^\prime(x)\text{,}\)

D. … is a function \(\ds F\) such that \(F(x)=f(x)\text{,}\)

E. … is a function \(\ds F\) such that \(F"(x)=f(x)\)

Answer
A.
36.

A critical number of a function \(f\) is a number \(c\) in the domain of \(f\) such that …

A. … \(\ds f^\prime(c)=0\text{,}\)

B. … \(\ds f(c)\) is a local extremum,

C. … either \(\ds f^\prime(c)=0\) or \(f^\prime(x)\) is not defined,

D.… \(\ds (c,f(c))\) is an inflection point,

E. … we can apply the Extreme Value Theorem in the neighbourhood of the point \(\ds (c,f(c))\)

Answer
C.

Answer the following questions. You need not show work for this section.

37.

What is the period of \(f(x)=\tan x\text{?}\)

Answer
\(\pi\text{.}\)
38.

What is the derivative of \(f(x)=x\ln |x| -x\text{?}\)

Answer
\(f'(x)=\ln |x|\text{.}\)
39.

If \(y=\sin ^2 x\) and \(\ds \frac{dx}{dt}=4\text{,}\) find \(\ds \frac{dy}{dt}\) when \(x=\pi\text{.}\)

Answer
0. \(\ds \frac{dy}{dt}=2\sin x\cdot \cos x\cdot \frac{dx}{dt}\text{.}\)
40.

What is the most general antiderivative of \(f(x)=2xe^{x^2}\text{?}\)

Answer
\(F(x)=e^{x^2}+C\text{.}\)
41.

Evaluate \(\ds \lim _{t\to \infty }(\ln (t+1)-\ln t)\text{?}\)

Answer
\(0\text{.}\) \(\ds \lim _{t\to \infty }\ln \frac{t+1}{t}\text{.}\)
42.

Does differentiability imply continuity?

Answer
Yes.
43.

Convert the Cartesian equation \(x^2+y^2=25\) into a polar equation.

Answer
\(r=5\text{.}\)
44.

Simplify \(\cosh ^2x-\sinh ^2x\text{.}\)

Answer
1.

Give an example for the each of the following:

45.

Function \(F=f\cdot g\) so that the limits of \(F\) and \(f\) at \(a\) exist and the limit of \(g\) at \(a\) does not exist.

Answer
\(\ds F=x\cdot \sin \frac{1}{x}\) and \(a=0\text{.}\)
46.

Function that is continuous but not differentiable at a point.

Answer
\(f(x)=|x|\text{.}\)
47.

Function with a critical number but no local maximum or minimum.

Answer
\(f(x)=x^3\text{.}\)
48.

Function with a local minimum at which its second derivative equals 0.

Answer
\(f(x)=x^4\text{.}\)

Answer the following.

49.

State the definition of the derivative of function \(f\) at a number \(a\text{.}\)

Answer

The derivative of function \(f\) at a number \(a\text{,}\) denoted by \(f^\prime(a)\text{,}\) is \(\ds f'(a)= \lim _{h \to 0} \frac{f (a+h)- f (a)}{h}\) if this limit exists.

50.

State the definition of a critical number of a function.

Answer

A critical number of a function \(f\) is a number \(c\) in the domain of \(f\) such that \(f'(c)=0\) or \(f'(c)\) does not exist.

51.

State the Extreme Value Theorem.

Answer

If \(f\) is continuous on a closed interval \([a, b]\text{,}\) the \(f\) attains an absolute maximum value \(f(c)\) and an absolute minimum value \(f(d)\) at some numbers \(c\) and \(d\) in \([a, b]\text{.}\)

52.

Match the start of each definition/theorem \(a - m\) with its conclusion \(i - ix\text{.}\) If there is no match, please state “no match”.

  1. The Mean Value Theorem states that …

  2. The chain rule states that …

  3. A critical number is a number that …

  4. The Extreme Value Theorem states that …

  5. Fermat's Theorem states that

  6. An antiderivative of a function \(f\) is …

  7. The natural number \(e\) is …

  8. An inflection point is a point …

  9. The derivative of a function \(f\) at a number \(a\) is …

  10. L'Hospital's Rule states that …

  11. The Intermediate Value Theorem states that …

  12. A function \(f\) is continuous at a number \(a\) …

  13. The Squeeze Theorem states that …

  1. … if \(f\) is continuous on the closed interval \([a,b]\) and let \(N\) be any number between \(f(a)\) and \(f(b)\text{,}\) \(f(a)\not= f(b)\text{.}\) Then there exists a number \(c\) in \((a,b)\) such that \(f(c)=N\text{.}\)

  2. … if \(f\) is a function that satisfies the following hypotheses:

    1. \(f\) is continuous on the closed interval \([a,b]\text{.}\)

    2. \(f\) is differentiable on the open interval \((a,b)\text{.}\)

    Then there is a number \(c\) in \((a,b)\) such that \(\displaystyle f^\prime (c)=\frac{f(b)-f(a)}{b-a}\text{.}\)

  3. … \(\displaystyle f^\prime (a)=\lim _{h\to 0}\frac{f(a+h)-f(a)}{h}\) if this limit exists.

  4. … If \(f\) is continuous on a closed interval \([a,b]\text{,}\) then \(f\) attains an absolute maximum value \(f(c)\) and an absolute minimum value \(f(d)\) at some numbers \(c,d\in [a,b]\text{.}\)

  5. … is in the domain of \(f\) such that either \(f^\prime (c)=0\) or \(f^\prime (c)\) does not exist.

  6. … on a continuous curve where the curve changes from concave upward to concave downward or from concave downward to concave upward.

  7. … the base of the exponential function which has a tangent line of slope \(1\) at \((0,1)\text{.}\)

  8. … If \(f\) and \(g\) are both differentiable then \(\displaystyle \frac{d}{dx}\left[ f(g(x))\right] =f^\prime (g(x))\cdot g^\prime (x)\text{.}\)

  9. … If \(f(x)\leq g(x)\leq h(x)\) and \(\ds \lim_{x\to a}f(x)=\lim_{x\to a}h(x)=L\) then \(\lim_{x\to a}g(x)=L\text{.}\)

Answer
  1. \(\displaystyle ii. \)

  2. \(\displaystyle viii. \)

  3. \(\displaystyle v. \)

  4. \(\displaystyle iv. \)

  5. no match.

  6. no match.

  7. \(\displaystyle vii. \)

  8. \(\displaystyle vi. \)

  9. \(\displaystyle iii.\)

  10. no match.

  11. \(\displaystyle i.\)

  12. no match.

  13. \(\displaystyle ix. \)